Thursday, December 27, 2018

How the LHC may spell the end of particle physics

The Large Hadron Collider (LHC) recently completed its second experimental run. It now undergoes a scheduled upgrade to somewhat higher energies, at which more data will be collected. Besides the Higgs-boson, the LHC has not found any new elementary particle.

It is possible that in the data yet to come some new particle eventually shows up. But particle physicists are nervous. It’s not looking good – besides a few anomalies that are not statistically significant, there is no evidence for anything out of the normal. And if the LHC finds nothing new, there is no reason to think the next larger collider will. In which case, why build one?

That the LHC finds the Higgs and nothing else was dubbed the “nightmare scenario” for a reason. For 30 years, particle physicists have told us that the LHC should find something besides that, something exciting: a particle for dark matter, additional dimensions of space, or maybe a new type of symmetry. Something that would prove that the standard model is not all there is. But this didn’t happen.

All those predictions for new physics were based on arguments from naturalness. I explained in my book that naturalness arguments are not mathematically sound and one shouldn’t have trusted them.

The problem particle physicists now have is that naturalness was the only reason to think that there should be new physics at the LHC. That’s why they are getting nervous. Without naturalness, there is no argument for new physics at energies even higher than that of the LHC. (Not until 15 orders of magnitude higher, which is when the quantum structure of spacetime should become noticeable. But energies so large will remain inaccessible for the foreseeable future.)

How have particle physicists reacted to the situation? Largely by pretending nothing happened.

One half continues to hope that something will show up in the data, eventually. Maybe naturalness is just more complicated than we thought. The other half pre-emptively fabricates arguments for why a next larger collider should see new particles. And a few just haven’t noticed they walked past the edge of the cliff. A recent report about Beyond the Standard Model Physics at the LHC, for example, still iterates that “naturalness [is] the main motivation to expect new physics.”

Regardless of their coping strategy, a lot of particle physicists probably now wish they had never made those predictions. Therefore I think it’s a great time to look at who said what. References below.

Some lingo ahead: “eV” stands for electron-Volt and is a measure of energy. Particle colliders are classified by the energy that they can test. Higher energy means that the collisions resolve smaller structures. The LHC will reach up to 14 Tera electron Volt (TeV). The “electroweak scale” or “electroweak energy” is typically said to be around the mass of the Z-boson, which is about 100 Giga-electron Volts (GeV), ie a factor 100 below what the LHC reaches.

Also note that even though the LHC reaches energies up to 14 TeV, it collides protons, and those are not elementary particles but composites of quarks and gluons. The total collision energy is therefore distributed over the constituent particles, meaning that constraints on the masses of new particles are below the collision energy. How good the constraints are depends on the expected number of interactions and the amount of data collected. The current constraints are typically at some TeV and will increase as more data is analyzed.

With that ahead, let us start in 1987 with Barbieri and Giudice:
“The implementation of this “naturalness” criterion, gives rise to a physical upper bound on superparticle masses in the TeV range.”
In 1994, Anderson and Castano write:
“[In] the most natural scenarios, many sparticles, for example, charginos, squarks, and gluinos, lie within the physics reach of either LEP II or the Tevatron”
and
“supersymmetry cannot provide a complete explanation of weak scale stability, if squarks and gluinos have masses beyond the physics reach of the LHC.”
LEP was the Large Electron Positron collider. LEP1 and LEP2 refers to the two runs of the experiment.

In 1995, Dimopoulous and Giudice tell us similarly:
“[If] minimal low-energy supersymmetry describes the world with no more than 10% fine tuning, then LEP2 has great chances to discover it.”
In 1997, Erich Poppitz writes:
“Within the next 10 years—with the advent of the Large Hadron Collider—we will have the answer to the question: “Is supersymmetry relevant for physics at the electroweak scale?””
On to 1998, when Louis, Brunner, and Huber tell us the same thing:
“These models do provide a solution to the naturalness problem as long as the supersymmetric partners have masses not much bigger than 1 TeV.”
It was supposed to be an easy discovery, as Frank Paige wrote in 1998:
“Discovering gluinos and squarks in the expected mass range [...] seems straightforward, since the rates are large and the signals are easy to separate from Standard Model backgrounds.”
Giudice and Rattazzi in 1998 emphasize that naturalness is why they believe in physics beyond the standard model:
“The naturalness (or hierarchy) problem, is considered to be the most serious theoretical argument against the validity of the Standard Model (SM) of elementary particle interactions beyond the TeV energy scale. In this respect, it can be viewed as the ultimate motivation for pushing the experimental research to higher energies.”
They go on to praise the beauty of supersymmetry: “An elegant solution to the naturalness problem is provided by supersymmetry...”

In 1999, Alessandro Strumia, interestingly enough, concludes that the LEP results are really bad news for supersymmetry:
“the negative results of the recent searches for supersymmetric particles pose a naturalness problem to all ‘conventional’ supersymmetric models.”
In his paper, he stresses repeatedly that his conclusion applies only to certain supersymmetric models. Which is of course correct. The beauty of supersymmetry is that it’s so adaptive it evades all constraints.

Most particle physicists were utterly undeterred by the negative LEP results. They just moved their predictions to the next larger collider, the TeVatron and then the LHC.

In 2000, Feng, Matchev, and Moroi write:
“This has reinforced a widespread optimism that the next round of collider experiments at the Tevatron, LHC or the NLC are guaranteed to discover all superpartners, if they exists.”
(NLC stands for Next Linear Collider, which was a proposal in early 2000s that has since been dropped.) They also iterate that supersymmetry should be easy to find at the LHC:
“In contrast to the sfermions, gauginos and higgsinos cannot be very heavy in this scenario. For example … gauginos will be produced in large numbers at the LHC, and will be discovered in typical scenarios.”
In 2004, Stuart Raby tries to say that naturalness arguments already are in trouble:
“Simple ‘naturalness’ arguments would lead one to believe that SUSY should have been observed already.”
But of course that’s just reason to consider not-so-simple naturalness arguments.

In the same year, Fabiola Gianotti bangs the drum for the LHC (emphasis mine):
“The above [naturalness] arguments open the door to new and more fundamental physics. There are today several candidate scenarios for physics beyond the Standard Model, including Supersymmetry (SUSY), Technicolour and theories with Extra-dimensions. All of them predict new particles in the TeV region, as needed to stabilize the Higgs mass. We note that there is no other scale in particle physics today as compelling as the TeV scale, which strongly motivates a machine like the LHC able to explore directly and in detail this energy range.”
She praises supersymmetry as “very attractive” and also tells us that the discovery should be easy and fast:
“SUSY discovery at the LHC could be relatively easy and fast… Squark and gluino masses of 1 TeV are accessible after only one month of data taking… The ultimate mass reach is up to ∼ 3 TeV for squarks and gluinos. Therefore, if nothing is found at the LHC, TeV-scale Supersymmetry will most likely be ruled out, because of the arguments related to stabilizing the Higgs mass mentioned above.”
In 2005, Arkani-Hamed and Savas Dimopolous have the same tale to tell:
“[Ever] since the mid 1970’s, there has been a widely held expectation that the SM must be incomplete already at the ∼ TeV scale. The reason is the principle of naturalness… Solving the naturalness problem has provided the biggest impetus to constructing theories of physics beyond the Standard Model...”
Same thing with Feng and Wilczek in 2005:
“The standard model of particle physics is fine-tuned… This blemish has been a prime motivation for proposing supersymmetric extensions to the standard model. In models with low-energy supersymmetry, naturalness can be restored by having superpartners with approximately weak-scale masses.”
Here is John Donoghue in 2007:
“[The] argument against finetuning becomes a powerful motivator for new physics at the scale of 1 TeV. The Large Hadron Collider has been designed to find this new physics.”
Michael Dine who, also in 2007, writes:
“The Large Hadron Collider will either make a spectacular discovery or rule out supersymmetry entirely.”
And Howard Baer in 2009:
“quadratic divergences associated with the scalar sector require new physics at or around the electroweak scale.”
The same story, that new physics needs to appear at around a TeV, has been repeated in countless talks and seminars. A few examples. Here is Peter Krieger in 2008:


Michelangelo Mangano:



Joseph Lykken:



I could go on, but I hope this suffices to document that pretty much everyone

    (a) agreed that the LHC should see new physics besides the Higgs, and
    (b) they all had the same reason, namely naturalness.

In summary: Since the naturalness-based predictions did not pan out, we have no reason to think that the remaining LHC run or an even larger particle collider would see any new physics that is not already explained by the standard model of particle physics. A larger collider would be able to measure more precisely the properties of already known particles, but that is arguably not a terribly exciting exercise. It will be a tough sell for a machine that comes at $10 billion and up. Therefore, it may very well be that the LHC will remain the largest particle collider in human history.



Bonus: A reader submits this gem from David Gross and Ed Witten in the Wall Street Journal, anno 1996:
“There is a high probability that supersymmetry, if it plays the role physicists suspect, will be confirmed in the next decade. The existing accelerators that have a chance of doing so are the proton collider at the Department of Energy’s Fermi Lab in Batavia, Ill., and the electron collider at the European Center for Nuclear Research (CERN) in Geneva. Last year’s final run at Fermi Lab, during which the top quark was discovered, gave tantalizing hints of supersymmetry.”

143 comments:

  1. "Since the naturalness-based predictions did not pan out, we have no reason to think that the remaining LHC run or an even larger particle collider would see any new physics that is not already explained by the standard model of particle physics. A larger collider would be able to measure more precisely the properties of already known particles, but that is arguably not a terribly exciting exercise. It will be a tough sell for a machine that comes at $10 billion and up. Therefore, it may very well be that the LHC will remain the largest particle collider in human history."

    Assuming that your arguments against naturalness are correct, they are much helped by actual experimental evidence, i.e. the non-discovery of particles predicted to be discovered based on naturalness arguments. So a more powerful collider would make your arguments more convincing. :-)

    I think that the whole idea of building something in order to confirm an expectation is not a good justification. Either we find something new or we don't. If we don't, then that is also valuable information. (There is a huge problem that negative results don't attract enough attention.) If we do, then it will be interesting---even more interesting since if it wasn't expected.

    Serendipitous discoveries have always happened. We know that elementary-particle physics is not complete, but we don't know what the missing pieces are. Perhaps serendipitous discoveries will help. As Rabi remarked when the muon was discovered, "Who ordered that?" To paraphrase Pirsig, the T.V. scientist who says "Our experiment is a failure; we didn't find what we were looking for." is suffering mainly from a bad script-writer. The whole point of an experiment is new information. If we knew precisely what we would find, then there is no reason to do the experiment---not the reverse.

    And yes, measuring known properties more precisely can be interesting; it can also be a hint of something we don't understand. QED is as well respected as it is precisely because well known quantities were measured more and more precisely. On the other hand, other measurements could indicate a departure from expectations, like the original Lamb-shift measurement. In the words of Dyson: "Those years, when the Lamb shift was the central theme of physics, were golden years for all the physicists of my generation. You [Lamb] were the first to see that this tiny shift, so elusive and hard to measure, would clarify our thinking about particles and fields." We should consider ourselves lucky that people didn't assume it wasn't there just because Dirac didn't predict it.

    ReplyDelete
    Replies
    1. The real problem lies in the unfulfilled promises. I don't think any theorist made those promises in bad faith, but they should at least refrain now from making the same promises while simply rehashing the very same flawed arguments. We cannot continue to believe that politicians will be easily fooled over and over again.

      Delete
  2. Phillip,

    Indeed, some physicists are arguing now that we should build a larger collider to test naturalness (or absence thereof, respectively). You have to love the irony in this: They want to test just how bad their predictions are.

    In any case, as I explain in my book, we cannot make all experiments we would like to make and therefore have to be careful where to invest our money. For this reason we need predictions that are reliable, so that we make experiments that are likely to find new clues that will allow us to develop new theories which will give rise to new technologies - and so on. We have broken this cycle of progress by banking on beauty-based predictions. Time to wake up.

    ReplyDelete
  3. It would be useful to measure when the temperature is sufficient to become a quark-gluon plasma and to know it's density as well as the temperatures and densities at denser ranges.

    ReplyDelete
  4. Great post. I hope the people quoted in this article read this blogspot and would be interesting to see their reaction.

    ReplyDelete
  5. Hi Sabine,

    very nice post, many thanks.

    How long, in your estimate, before some physicists begin to suggest that particles and fields are not the fundamental stuff? If that is true, how long before they begin to understand what it is?

    Thanks,
    J.

    ReplyDelete
  6. akidbelle,

    There are many such predictions. String theory is one of them. I mention some others in my book.

    ReplyDelete
  7. bee:

    "How the LHC may spell the end of particle physics". and, as the idiom goes 'Good riddance to bad rubbish.' [and to bad science].

    btw - a new book "Quantum Space" (available at Amazon UK now and at Amazon in a few weeks) is a really superb book on LQG. It is a 'pop science' book only in the sense of being equation-free. it traces the history of the development of LGQ (and string theory as well) and provides relevant biographical details on Rovelli and Smolin. it is by far the most impressive book on 'foundational physics' (as it is 'modestly' referred to by those who work on quantum gravity). i hope you will review this book with the usual incisiveness that you use in discussing arXiv manuscripts.

    naive theorist

    ReplyDelete
  8. naivetheorist,

    I am reading "Quantum Space" as we speak!

    ReplyDelete
  9. "In any case, as I explain in my book, we cannot make all experiments we would like to make and therefore have to be careful where to invest our money. For this reason we need predictions that are reliable, so that we make experiments that are likely to find new clues that will allow us to develop new theories which will give rise to new technologies - and so on."

    A collider, like a telescope, is a rather generic instrument. Many big instruments were pitched with the hope (well founded or not) of doing this and that, but earned their keep by discovering other things no-one had even thought of. Pulsars are an example of a serendipitous discovery. So was the CMB (it had been predicted, but that didn't influence the discovery.) Yes, choices have to be made, but it is cheap in the grand scheme of things. As Sagan said justifying the cost of space probes, "a penny a world for each person on Earth". And in any case, new technologies (spin-offs etc) are not the goal of basic research.

    ReplyDelete
  10. https://www.youtube.com/watch?v=yKeHQpT5wVE
    … Reroute all pissy national research budgets into growing a Chinese hypercollider - four times the currently planned circumference, Chinese supercomputing hardware - and have done with it.

    ReplyDelete
  11. Governments in the G20 that support this type of research are in fact going bankrupt. As the post WWII generations (the Baby Boom) reach old age, costs for pensions, medical care, and custodial care will skyrocket, while the number of prime age workers who pay taxes to support those costs collapses.

    Given the choice between funding research in physics and paying old age costs, physics will lose. So will a lot of other stuff, but investments without a guaranteed immediate return just won't be made.

    Get used to it people.

    ReplyDelete
  12. Bee,

    is 100 TEV collider high enough of an improvement over 14TEV? and if the 100 TEV doesn't find anything what is the next energy frontier to aim for?

    I know this is going to be expensive but instead of aiming for 100TEV, why not 200TEV or 300TEV or even 1 Peta EV collider.

    why spend $20 billion on a 100TEV collider when for another $10 billion you can get perhaps a 200TEV collider or $40 billion for a Peta-EV scale collider

    ReplyDelete
  13. Philip

    Your argument about null data is of course absolutely correct; not finding something teaches us about the nature of the universe too, however what you don’t take into account is human nature and behavior, something I’ve observed and understand very well. I will predict with confidence that Sabine’s conclusion is correct, that instead of admitting defeat the vast majority will speculate, pontificate, and hypothecate why they are still right and the answers to prove it are one more collider or new experiential apparatus away. Those resources would very likely produce much more good science in other areas of research in lieu of yet again confirming a null result (the most likely outcome).

    ReplyDelete
  14. The kindle version of JB’s Quantum Space is available now in the US, on sale for $10...

    ReplyDelete
  15. I am on the fence when it comes to the proposal for a higher energy collider. On the one hand it might probe new physics if it is there, but there is less reason to think there is new physics at this scale than in the past.

    We of course can't do anything with quantum gravity. Analysis of data from LIGO puts a pretty negative spotlight on the idea of large extra dimensions where gravity might “leak into.” Read https://phys.org/news/2018-09-gravitational-dose-reality-extra-dimensions.html and references for more. So we have little reason to think quantum gravity or stringy physics should show up with a new collider that is a small multiplier over the LHC. Quantum gravitation occurs at a scale of 10^{14} to 10^{16}TeV and the 10 TeV work we can do is 13 or more orders of magnitude too low. To get quantum gravitation this way would be to think we could get Higgs physics with the study of 1ev physics of ionized atoms etc.

    For quantum gravitation we need to let nature do the heavy lifting. Black hole collisions may give data on quantum hair, or as Mathur has it with “fuzz,” on stretched horizons. Black hole collisions are in a way quantum gravity collider experiments that nature provides for free. We have only to invest in the detectors. With the eLISA we might be able to measure the BMS symmetries associated with black hole quantum hair.

    Even if we have success with this we may still be faced with the problem of how to get lower energy physics up to the TeV region to link with quantum gravitation. It is possible in the sense of swampland such connections may fundamentally not exist.

    ReplyDelete
  16. I think the direction of particle physics should be towards neutrino physics and precision tests. There are really 3-5 communities in particle (and astro-particle) physics and the collider community is only one (although by far the largest and strongest). There are a lot of exciting results to come out of neutrinos which we know are there, similar to the Higgs, and we can use them to better construct BSM models. A lot of money and momentum is with the traditional collider program and even in neutrinos people want to get the big results (CP violating phase) now and not wait and do the necessary Research and Development to improve the technology for better neutrino beams. There are a couple of ways forward, one that was being pursued before (a neutrino factory based on a muon accelerator, with equal amounts of muon and anti-electron neutrinos) and one suggested by a recent analysis of MiniBooNe where it was observed that the decay of Kaons at rest creates a mono-energetic neutrino 'beam' of 236 MeV.

    ReplyDelete
  17. "Besides the Higgs-boson, the LHC has not found any new elementary particle."



    So will the various taxpayers get their money back, minus the value of the Higgs confirmation? I'd love to hear particle physicists discuss that value with the gilets jaunes.

    Here in the United States, I think we have at least half a billion 'invested.' Maybe we could have physicists teach our children to actually be able to compute unit pricing in their heads at the supermarket to pay off the debt. The result would be of far greater value than the 'discovery' of something that everyone who knows already agreed existed.

    I'm a fan of science since the days of the Mercury missions, and I spent five years in grad school studying genetics and evolution, but that doesn't make me stupid. This whole project was a con game from the start. Governments saw it as a jobs program for brainiacs, and a manufacturing technology driver - politicians who say 'new-kwu-lar' for nuclear certainly don't know a boson from botox.

    Let me suggest that if it costs this much money to learn about the nature of the universe, we can get by just fine with the knowledge we have already. Lesser amounts of money can pay off with far more sure knowledge - a world-wide exploration of the bottom of the seas, anybody? Unlike Mars, we know there's life there.

    P.S. Just read the book - exactly what I enjoy best about science writing. A window into a world I would never otherwise know. I've recommended it to my local library.

    ReplyDelete
  18. High energy physicists were lucky that at least one particle was waiting for the LHC to discover. If they successfully lobby for a more powerful machine, and it finds absolutely nothing, that really will be the end!

    ReplyDelete
  19. Chiral molecule microwave rotational spectrum probes of vacuum microsymmetry breaking, Sakharov criteria then baryogenesis, run ~1-3 GHZ, or 0.41 to 1.2×10^(-5) eV. The past 40 years of increasingly desperate speculation denies trace imperfection.

    This net matter universe exists. Physics expansively expensively insists it does not. Look the other way.

    ReplyDelete
  20. I wonder, How can a referee accept one of your papers having all the main physicists against you? (Specially after this article). The status quo is hard, it can leave you out of the system.

    ReplyDelete
  21. So, lets assume we do NOT build the next collider. What, in Your opinion, would be the best target(s) to channel those billions of EUR to? Gravitational astronomy?

    ReplyDelete
  22. I blame Trisolarians for the lack of discoveries.

    But the question is, what experiments can we do apart from bigger colliders? A space-based gravitational wave observatory?

    ReplyDelete
  23. You are truly fearless, Sabine.
    Being confronted with past statements one would rather forget should be an occasion for self-reflection, but I am afraid this will not be taken as such an occasion by everyone. Physicists are only human, too. I do hope that people will listen to your arguments and consider them carefully and dispassionately.

    ReplyDelete
  24. Dear Sabine,
    I cannot find information about generation of any positively charged particles in collisions of two electron beams (not electron-positron, but electron-electron beams). I am curious if such experiments were done when LEP collider was still in work. And, will standard model predict generation of any positively charged particles in a collision of two electron beams? If standard model predicts such events and there is no experimental data, why not to do such an easy CONTROL experiment ?

    ReplyDelete
  25. Quantum Space....when I search all I get is fiction book. Is that correct?

    ReplyDelete
  26. Calling a next-generation collider a "tough sell" is an understatement.

    I'm a semi-educated layman who's vitally interested in fundamental scientific questions. But my take on the LHC is this: after decades of work, at a cost of many billions of dollars, it produced the number 125. And that looks to be about it.

    I understand the importance of basic research that may not pay off for decades; and I get the value of the "null result". But if I were administering a bit research budget, and were asked to choose between going in on a bigger collider or funding, say, a new idea on carbon capture from the atmosphere - it would be a no-brainer. I know which project I'd like to put in front of my trustees, reagents, or board of directors.

    ReplyDelete
  27. Sabine...

    You hang out with theorists too often.

    Most of the experimentalists I know hoped we would find something at the LHC. There was obviously some tendency to look for SUSY, as the theorists loved it, but most of us were just looking for "something," where something is simply defined as "not standard model." Personally, I was hoping for something to explain the generation problem...perhaps compositeness or some other equally disreputable idea that theorists didn't like. But the certainty that we would find SUSY was not a widespread illness of the experimental community.

    I do think it's a fair question to ask whether a 100 TeV machine is where to look. One could also talk about precise measurements of the electron dipole moment, or things like that. My current hopes are on a discovery by g-2. I'm not sanguine about the prospects of an LHC discovery, except for something that is currently limited by the amount of data, which LHC run 3 and 4 will help with.

    I did read your book and understand your points. They were largely correct, but the illnesses you describe were not pervasive in the experimental community.

    For whatever it's worth, I predict that the next big discovery will be something that was not popular with the theoretical community, but we'll find that some obscure paper from the 1990s predicted it.

    But, with all that said and with your disdain for the current theoretical approaches that are popular...and I understand why you hold them...what would >>YOU<< do? We're now in a magic world. I have $20 billion. I live in a world in which we need to maintain a community of thousands of technically adroit people that need to be kept busy, or they will disperse into industry and it will take 20 years to rebuild the expertise if we don't keep the expertise current. We need to have a higher-likelihood project to pursue for the next 30 years.

    I put you in charge of deciding how to move the community forward. You have carte blanche. What would >>YOU<< do?

    ReplyDelete
    Replies
    1. It will be very hard for any theoretician to acknowledge the fact that without empirical evidence supporting their ideas these ideas are mostly irrelevant; this impasse in HE theoretical physics clearly shows that what is needed is more empirical data no more empty "predictions"; but this impasse will go on as far as the "leaders" of this community are exactly the same theoreticians that had promoted and then milked "fairly tale physics" ideas far detached from Reality, post-empirical Science being the obvious oxymoron.

      For these theoreticians it will be very hard to acknowledge that experimental physicists do not "need" their "guidance" to serendipitously find new things; acknowledging that will take away their "leadership".

      The history of Science and even relatively recent results in complexity theory strongly point to the idea that "new" things always will be found, that the narrow wishful thinking in assuming that our knowledge of Reality is "complete" or near to be completed is shortsighted; new independent/emergent properties are ubiquitous and any theory always will be limited by its "complexity": its results cannot be more complex than the theory itself(Chaitin heuristic principle).

      Nothing can replace the constant observation and testing of Reality; we need a lot more physicists doing experiments and a lot less armchair experts guiding and defining the budget assigned to these experiments.

      Delete
  28. @ neo,

    The largest collider we could in principle build is one that circles the Earth. The Earth is about 6400km in radius or 40200 km in circumference. The LHC is 27 km in circumference so this putative collider would be 1500 times the circumference and about as much larger in energy. This would then have energy up to about 19500 TeV or around 2.6x10^{-12} the Planck energy. So quantum gravity is not in its range by a long shot. Of course there might be unknown things in this energy range. A Planck scale collider would encircle the entire Milky Way galaxy. I doubt an Earth scale collider will ever be built.

    Of course there is miniaturization. RF cavities for particle accelerators that employ EM waves could be replaced with optical systems with a 10^6 or compression of size or a corresponding increase in energy. There are schemes for using lasers to accelerate particles in channeling or plasmas. Maybe an LCH energy machine could be made to sit in a single building. This might be the best technical prospect for jacking the energy of colliders to much higher energy.

    ReplyDelete
  29. Bee, I believe the LHC was built to discover/rule-out the Higgs and not to discover SUSY; this was a minor PR bonus. The LHC will now probe the properties of the Higgs marking the beginning of a new era in particle physics and Higgs factories :)

    ReplyDelete
  30. The biggest weakness of naturalness is that it is natural. My supernatural theory of everything will show how. However supercolliders will continue to be useful to strengthen the standard model and to search for physical realities that will take physics towards understanding what underpins quantum theory and mechanics. There are unknown facets of the interrelationship between particles and spacetime. We may also more profoundly probe the neutrinos in interactive ways with other realities.

    ReplyDelete
  31. In my forties, I suddenly got as interested in economics as science. I'm always concurrently reading science and economics books.

    Sabine has given us a list of predictions made by scientists. One difference between economists and scientists is that economists will sometimes take credit for correct predictions when they don't deserve credit. An economist might say that a nation's economy will get better or worse because of some policy, but it's often not that straightforward. Many economists don't want to admit that some of their correct predictions are more a matter of random chance and less a matter of accurate models. As Friedrich Hayek said, "The curious task of economics is to demonstrate to men how little they really know about what they imagine they can design."

    Particle physicists want to design new colliders. When scientists make incorrect predictions, they might make excuses (and ask for bigger colliders), and they might point out that it's just as important to be proven wrong as to be proven right. Being proven wrong is an essential part of scientific methodology, so scientists have some cover that economists don't have.

    The general public is aware that scientists and economists make mistakes, even if scientists and economists don't admit it. The financial crisis of 2007/2008 has done irreparable damage to the public's faith in economists. In the US, each time we elect a Republican president, an army of economists tell us that huge cuts in taxes and regulations are the key to prosperity.

    The American public is equally skeptical about what scientists tell us about our health. For decades it's been a running joke that scientists can't make up their minds about which foods are good and bad. The public is equally confused about fluoride or chlorine in the water, about cellphones causing cancer, or about smartphones making our kids dumb and socially/emotionally inept.

    For the majority of the public, colliders aren't on the radar. Most people know that colliders exist, but they have only a vague understanding of what scientists are trying to do with them. If you were to ask a random person about colliders, you'll occasionally find one who knows that scientists are using them to understand the fundamental nature of matter and the physical laws of nature. The general public still has faith in the potential benefits of scientific breakthroughs.

    That's really what it comes down to. If the general public supports colliders, it's mostly because they think that scientific breakthroughs lead to practical applications. A smaller part of the public might support fundamental scientific research purely for the sake of curiosity.

    All the people I know who support a manned mission to Mars by 2035 say it's worth it for the sake of fun, adventure, exploration, etc. A large part of the fun is how challenging it would be. But because I study economics, I noticed that they don't realize that having that kind of fun has a huge opportunity cost. If I mention cost, I'll typically get the same logic as Roberto offers in his defense of building a new collider: It's not an expensive project if you look at the per capita costs. If I persist, they'll drop the fun part and retreat to safer ground: A manned mission to Mars will almost certainly give us lots of nifty technological spinoffs.

    So I'll conclude by pointing out that this is where science and economics intersect. For better and worse, scientific projects are largely justified by economic arguments. When scientists ask political leaders to fund their projects, they mention potential economic benefits, practical applications or spinoffs. They never just leave it at pure curiosity and fun.

    Maybe in the future we'll have the luxury of doing expensive scientific research purely to satisfy our curiosity, when we no longer have to worry about scarce resources. To my way of thinking, that would be a reasonable version of utopia.

    ReplyDelete
  32. It's a bad definition (just ask a musician or athlete) but it is commonly held that: The definition of insanity is doing the same thing over and over and expecting different results.

    (In fact, it's a motto from twelve-step programs in reference to addictive behavior, so perhaps it applies all the more.)

    ((Which reminds me: There is one piece of evidence in favor of SUSY. The snark particle was discovered shortly after the internet went online...))

    (((Merry Christmas! And A Happy New Year!))

    ReplyDelete
  33. Don,

    That's right, my book is about theory-development. Glad to hear you didn't hate it.

    I sent you an email on Oct 20 about what I would do if I had money. You didn't reply.

    ReplyDelete
    Replies
    1. Well, your public also wants to know! Why hide it in an email to one person? Accepting the premise that colliders are a practical dead end, surely we can't all be expected to pick up our bats and balls and go home. What *is* promising?

      Delete
  34. Larry,

    That's true. It is also true that a lot of physicists very clearly left the public with the impression it would do more than that.

    ReplyDelete
  35. Dr. Hossenfleder is the Jon Stewart of Physics.

    ReplyDelete
  36. Seems, partcle physics are at late evening on July 13th, 1789?

    The Estates-General have failed and next day the Bastille may fall - or Empire strikes back...

    ReplyDelete
  37. One is reminded of the fallacies of the CAGW sell.

    ReplyDelete
  38. Equating science and economics is either naive or evil.

    ReplyDelete
  39. I love Dr. Hossenfelder. Academia is corrupted as an establishment, driven to seek vast public funding in order to keep the machinery of tenures, grants, department budgets going and growing. The prospect that it's all going to come to a halt for industry of particle physicists has got to be terrifying. If bogus reasons can be fabricated to keep the mill going and public funds flowing, you can expect many to try. Dr. Hossenfelder is an appropriately responsible scientist calling out the Particle Physics industry for the scam it's preparing to pull in arguing for a bigger collider. I can't express how much I like her and value her as a public voice for honest science shorn of institutional corruption.

    ReplyDelete
  40. I forgot to mention another way that particle physics could be done without digging supersized circular prairie dog towns in the ground. Recently evidence of a supersymmetric pair to tauon or the sτ slepton. I here reference the popular site ( https://bigthink.com/surprising-science/cosmic-rays-from-earth?rebelltitem=4#rebelltitem4 ) Cosmic ray research is the poor cousin or Rodney Dangerfield (I get no respect) of particle physics. However, it holds a potential for detecting particle physics up to 10^9 GeV.

    Since more of this is done in Antarctica I have thought the atmosphere there could act as a scintillation medium. An array of heliostats, long duration balloons, with detectors over Antarctica during the winter dark might then find the tracks of cosmic ray particles and daughter products over a huge area.

    The controls are not as good and the statistics are poorer, but at least the costs might be far lower. We could measure particle events at several orders of magnitude higher energy than we can ever muster with a collider.

    ReplyDelete
  41. From one who sits in the nose bleed rafters with a child like amazement of physics, I agree, time to stop the spending and concentrate on technologies that can help our fellow humans.

    ReplyDelete
  42. It turns out Don Lincoln did perfectly well receive my email from several months ago. He just repeats empty accusations because he thinks that makes him look smart.

    ReplyDelete

  43. Blogger Lawrence Crowell "Cosmic ray research...detecting particle physics up to 10^9 GeV"
    That is 10^6 TeV. Center-of-mass energy rather than lab frame, 1000 TeV. The Chinese supercollider is planned for 100 TeV p-p. If all the pissy little other future collider budgets were added, quadruple it, 400 TeV CoM.

    Now, consider luminosity versus 2.5× in CoM energy.

    ReplyDelete
  44. Lawrence,

    Uncle Al beat me to it. You have to distinguish the energy of the incoming particle in the Earth restframe (which is what you quote) from the center-of mass energy. Also, note that the flux on the highly energetic tail is miserably small. That, plus cosmic rays have a lot of uncertainty. To begin with, you don't know what the incident particle is. Next, the scattering doesn't happen in a vacuum, so you create showers that are difficult to simulate. Finally, you cannot cover the whole surface of the earth to capture all of the particles that were created. Particle colliders are without doubt a considerably cleaner signature. Best,

    B.

    ReplyDelete
  45. Unknown wrote: Equating science and economics is either naive or evil.

    Who equated the two things?

    From your tone, it looks like you have a very low opinion of economics. I'm curious about that, but since this is a physics blog, I won't ask.

    Unknown wrote: One is reminded of the fallacies of the CAGW sell.

    It seems like just about anything reminds you of the fallacies of the CAGW sell. You're the Julian Simon fan, right?

    ReplyDelete
  46. Sabine,

    That last sentence about Don is a bit harsh and I disagree. I generally support your assessments, I have noted Don to be a bit defensive at times however, I believe he is sincere in attempting to have a constructive dialogue about the issues he disagrees with you. These is no pretense in his arguments, yet like most of us he is mostly unaware how his feelings and self interests sway is beliefs. Something we all need to be more aware of.

    ReplyDelete
  47. Bee and Al,

    I agree cosmic rays are not certain or nearly as good. Also the spray of particles is narrowly focused as one is in a lab frame and not CM frame. However, any bets on getting a collider to 1000TeV or higher? As Napoleon put it, one starts out not with the army you want, but the army you have. Pressing forwards with particle physics far beyond the 10TeV level we currently work in may leave us with little choice.

    ReplyDelete
  48. @Don Lincoln: I don't understand the argument about "we need to build the next particle accelerator before all the people with expertise drift off into industry".

    If the LHC doesn't discover anything beyond the Higgs, I don't see why there would be a good reason to build another particle accelerator anywhere in their lifetimes, so why do we need to preserve their expertise. And if the LHC does discover something in the next few years, and we do start building another particle accelerator, surely enough of them would prefer building particle accelerators to their jobs in industry to come back.
    javascript:void(0)


    ReplyDelete
  49. What to do about it? In his book Inadequate Equilibria, Where and How Civilizations Get Stuck, ELIEZER YUDKOWSKY proposes a solution to get out of the bad Nash equilibrium caused by bad incentives. He suggests fresh organizational models:


    … A critical analogy between an inadequate system and an efficient market is this: even systems that are horribly inadequate from our own perspective are still in a competitive equilibrium. There’s still an equilibrium of incentives, an equilibrium of supply and demand, an equilibrium where (in the central example above) all the researchers are vigorously competing for prestigious publications and using up all available grant money in the course of doing so. There’s no free energy anywhere in the system. ...

    … I responded that we were a small research institute that sustains itself on individual donors, thereby sidestepping a set of standard organizational demands that collectively create bad incentives for the kind of research we’re working on. I described how we had deliberately organized ourselves to steer clear of incentives that discourage long-term substantive research projects, to avoid academia’s “publish or perish” dynamic, and more generally to navigate around the multiple frontiers of competitiveness where researchers have to spend all their energy competing along those dimensions to get into the best journals.
    These are known failure modes that academics routinely complain about, so I wasn’t saying anything novel or clever. The point I wanted to emphasize was that it’s not enough to say that you want risky long-term research in the abstract; you have to accept that your people won’t be at the competitive frontier for journal publications anymore.
    The response I got back was something like a divide-by-zero error. ...

    ReplyDelete
  50. Good post and excellent work. We, mostly heretics, could see this happening from back in the 70s. Thanks for pointing out some of the crackpots. Your book is a milestone in science.

    ReplyDelete
  51. Lawrence Crowell Bee, Don,

    wouldn't the simplest and least expensive path is after HL-LHC upgrade, to re-use the LHC tunnel, and simply replace the current 8 tesla magnets with 16 tesla magnets or better, and double from 14 TEV to 28 TEV or even 33 TEV?

    the only change is using more powerful magnets.

    ReplyDelete
  52. Louis,

    Don asked me several months ago for my research proposals. I sent him a brief, one paragraph summary, of one I thought he would be particularly interested in. A month later, he submits the same question again on this blog. I reply I already told him. Today, he submits the same question again. I resend my email. He responds that he received the email from months ago.

    I can only conclude that he is not remotely interested in my answer, and the reason he continues to ask this question nevertheless is that he wants to falsely raise the impression I have no own research projects.

    I have said this several times previously, but I will say it once again. The reason I do not publicly discuss my research projects is that there are many people who would jump at the opportunity to then try and ignore my arguments by claiming I am trying to promote my own research. This is what happened with Lee Smolin's book, for example. My interpretation of Don's behavior is that he tries exactly this. I am not amused.

    Yes, my response may be harsh. But I have yet to encounter a single particle physicists who, instead of attacking me personally, actually thinks about what I have told them, and I have no intention to be polite about the nonsense I have to endure.

    ReplyDelete
    Replies
    1. ...Sabine, which of Smolin's books were criticized thusly? Is it bad to have your own research interests / point of view?
      Unfortunately, as we reach a dearth of new (but wrongly anticipated) results, we are left to argue that theories can be excluded a priori... including otherwise good ones.

      Delete
    2. Instead of asking you to go into details of your own research, then, let me as a member of the lay public phrase a question that hopefully has a broader answer, then: if it looks like SUSY is dead, and bigger colliders are a waste of money until Kardashev 2 or so (that's tongue-in-cheek), what sort of general research program to further understand fundamental physics is worth taxpayer support?

      Delete
  53. I agree with Don. I don't know anyone in the experimentalist community who takes naturalness particularly seriously. Some of the theory community is certainly in a state of mourning akin, so I'm told, to the atmoshpere in the community when the first proton decay search results came in and refuted SU(5) GUTs.

    For the experimentalists, the LHC has been about the Higgs + SM tests and exploration of the new energy regime. Having collected 5% of the dataset to be delivered over the LHC's lifetime, we tend to see the experiments as having just begun. It certainly feels a bit strange to have the "nightmare scenario" declared before we even get into high luminosity running.

    Regarding naturalness, it doesn't matter how many of the great and good are quoted as saying they expected SUSY etc. at ~1 TeV. The naturalness/fine-tuning argument is by its very nature imprecise. All one can say is that the predicted physics from this argument is expected to occur around the TeV-scale. The TeV-scale extends beyond our current limits (up to ~2 TeV depending on the scenario). I have little desire to defend naturalness in the Higgs sector as I've never been particularly convinced by it. However, if one does buy it then a dispassionate assessment of its predictions leaves lots of room for as yet undiscovered new physics beyond the current limits.

    I don't care how many phenomenology models get killed and are then reborn with higher mass sparticles or how many theorists have voiced their opinions about the exact degree of fine tuning that Nature would accept. The "1 TeV and the LHC must see it" argument seems to be pretty much an example of groupthink no different to the others one sees around us in science.

    ReplyDelete
  54. Good article and congrats on your very engaging book. End of physics or end of particle physics or are they the same? Regret missing the next century. I think there are some “paradigm shifts” around the bend.

    ReplyDelete
  55. It looks like nobody has ever done collision of two electron beams in collider. And, nobody is interested to do this (of course no new particles - no papers and grants :)). It is theoretically trivial since at certain level of energy the electron-positron pairs should be generated. Ok, it is not interesting, but in science controls are obligatory and if the energy level for such events are reachable in modern colliders, why particle physics is exceptional science. I wonder, when no electron-positron pairs (or any positively and negatively charged particles pairs) will be detected after collision of two electron beams, how standard model will explain this?

    ReplyDelete
  56. Bee, your criticism is noted in media "these have come in for increasingly harsh criticism, and several researchers are now asserting that none of the models proposed to date are workable."
    Referring to string theories and SUSY.
    https://m.phys.org/news/2018-12-universe-extra-dimension.html although I don't think their DE idea will survive the empirical cuts made by the Dark Energy Survey.

    ReplyDelete
  57. Eugen (and anybody else searching for Quantum Space.
    In the UK the book is available online from Waterstone's.

    ReplyDelete
  58. @ Lawrence Crowell

    NASA’s Antarctic Impulsive Transient Antenna (ANITA) has detected an ultrahigh-energy particle with about 500 times the mass of a proton coming from the direction of the Antarctic ice.

    https://dailygalaxy.com/2018/10/reboot-new-evidence-suggests-mystery-particles-detected-beaming-up-from-south-pole-do-not-fit-the-standard-model-of-physics/

    The particle shower that the particle produces indicate that the particle had energies in excess of 0.5 exa-electron volts—70,000 times higher than the energy achieved with the most powerful particle accelerator.

    There are a number of puzzles that are coming from the ice environment. Pluto is geologically active and Ultima Thule might still be. Its hard to believe that the heat that is keeping Pluto geologically active is still coming from freezing ice after 4.5 billion years of freezing. It might be best to understand the mysteries that nature shows us when they reveal themselves.

    ReplyDelete
  59. Louis wrote: he is mostly unaware how his feelings and self interests sway his beliefs

    I have to chuckle a little, because you thought Sabine was a bit harsh, and then you say that Don is pretty bad at applying one of the essential principles of scientific methodology, namely, guarding against bias and striving to be objective. If he's "mostly unaware" of it, he's pretty bad at it. That's harsh, man. :-)

    Supreme Court justices had *better* be aware of how their feelings and self-interest can sway their decisions, and they had *better* guard against it. If everyone in a position of power were mostly unaware of how their feelings and self-interest can affect their beliefs, we might as well give up any hope for humanity. The fact that we've made it this far demonstrates that some people are aware. That's the good news. The bad news is that maybe not enough people are aware to avoid making a big mess of things.

    ReplyDelete
  60. Sabine wrote: I have no intention to be polite about the nonsense I have to endure

    One of my favorite lines from one of my favorite plays, The Importance of Being Earnest:

    "I think that whenever one has anything unpleasant to say, one should always be quite candid."

    ReplyDelete
    Replies
    1. With more apologies to Wilde, I never travel without my copy of Lost in Math. One should always have something sensational to read in the train.

      Delete
  61. Let the Chinese People to build their Guinness Records Collider ... Yep, very likely, they will not find new particles, but on a few decades, AI will find the techniques to sync colliders all around the World, then, The Apes will witnessing New Physical Phenomena related to entanglement and quantum thresholds for error correction.

    ReplyDelete
  62. Some of your commenters talk about a higher energy collider, and the importance of negative results from it, as though this was just a question of money. I wonder who is going to spend decades building these machines & detectors and then spend the rest of their lives operating them and analyzing the data. Especially when the prospects look more-and-more like more-and-more negative results.

    I started my career in experimental particle physics in the early sixties, when the experimental groups were small, with the attitude that there was some likelihood that I would be centrally involved in an important discovery. This passion for discovery was a characteristic of all my colleagues at that time, and drove the creativity of the field (~4 orders of magnitude improvements in accelerator/detector/data-analysis capabilities). Of course, in the intervening time, most of us didn't make a spectacular discovery, but, on the other hand, some people did; CP violation, neutral currents, J/psi, b-quark, neutrino oscillations etc, etc. So, discovery or not, for me it has been an incredibly exciting career. No regrets, none at all!

    I try to imagine how I would feel if my career were starting now, with the prospect of working in a multi-thousand person group, and ultimately spending years working on some analysis project with a >99.9% probability of ending in a negative result, and even then, when my (life's?) work is finally published (after it's rewritten by some committee) my name will be buried somewhere in the middle of the 7th page of a 10 page author list. Not for me, no way!

    If (when?) this next machine is built and running, it will involve people with very different mentalities than those of the cohort that got us to where we are today. It is not obvious to me that the impressive intellectual progress made during the past half-century will be sustained.

    ReplyDelete
  63. Summerisle,

    You're missing the point. The experimentalists who supposedly don't buy what those theorists say (which I have no doubt exist) are invisible to the public. None of them has ever been heard of. None of them have objected when some theorists made big proclamations. For what the public is concerned, particle physicists made predictions, they all agreed on those prediction, and those predictions were all wrong.

    I am telling you, your field has to clean up its business. If the head of Fermilab (who, for all I can tell, is an experimentalist) wrongly tells the BBC that we know there must be new science, I expect that particle physicists correct this statement. Yet, no one said a word.

    If the FCC working group at CERN puts out a video that, once again, makes the same false promises, I expect that particle physicists object and ask that the video be taken down. What's the reaction that I actually did get from particle physicists? They think it's okay to lie to the public. (And publicly say so.)

    I cannot recommend to finance people in this field as long as this is going on.

    ReplyDelete
  64. WRL,

    As much as I want to believe that someone is finally listening, I think that the sentence you quote refers to the problem that string theorists now think that we may be living in the "swampland", see eg here.

    Btw, a lot of people asked me to write about this. I will not. I don't want to waste my time with this. There is nothing to learn here other than that string theorists continue to cook up models that have nothing to do with reality and from which we learn nothing.

    ReplyDelete
  65. WRL,

    Almost all physicists ignored the criticism that Lee Smolin raised in "The Trouble of Physics" by proclaiming that he was attacking string theory to promote loop quantum gravity, a field he happens to be involved him himself.

    Now, look, I know Lee well enough to tell you that this is nonsense. To begin with, he hasn't been all that deeply tied into LQG for a long time (look at his publications) and also, he has demonstrably a larger interest in how science works (which he wrote about elsewhere). But really it didn't matter in the end.

    You see, that's the tragedy. Doesn't matter how good your arguments are, if people can ignore them, they will.

    (To be fair, I think the group-think is as bad in LQG as in string theory. It's really the same problem everywhere.)

    In any case, at a minimum I can try to not make the same mistake. It may not help, in the end, but at least I can try, so I will try.

    ReplyDelete
  66. Josh,

    For starters, I did not say that SUSY is dead and I did not say that larger colliders are a waste of money.

    With that ahead, you are asking a good question, but it's not one that I (or anyone else, for that matter) can answer simply by way of evaluating their own opinion. It requires that you do a careful cost/benefit analysis. For this, you not only need to know the projected expenses, you also need to know what's the potential payoff and how reliably this will come in. This is why I keep going on about the rotten predictions of particle physicists - you cannot trust those predictions and that is important information.

    In other words, you are asking for a simple answer where there are no simple answers. It's complicated.

    If you ask for my personal opinion, I explain in my book where I think breakthroughs are likely to come from. We should either focus on inconsistencies between experiment and theory that you already know of - and keep doing this until a new picture clicks in place - or rely on theoretical predictions based on internal inconsistencies. This is what has historically worked. Relying on arguments from beauty can work if we are lucky, but it's unlikely to work.

    In the foundations of physics, the only example of an inconsistency between theory and experiment is currently dark matter. Two examples of the latter are quantum gravity and the measurement problem in quantum foundations. There may be more, but the theory-development is so far behind that I can't tell.

    Those are the cases where I think you'll get the biggest bang for the buck if you want to invest into the foundations of physics. Particle colliders are the most expensive experiments there are, and in the present situation I don't think we'll get much out of it. Hence, I suspect that if you did do a careful cost/benefit analysis, particle colliders would not come out top.

    Having said this, if I got to make any recommendation for investing in science, I would recommend you implement measures that counteract the social reinforcement in scientific communities - and, trust me, I can go into great detail on how to do this. A lot of money is currently going to waste because we are paying people to do nonsense work. It's a very inefficient way to invest money.

    Best,

    B.

    ReplyDelete
  67. bee:

    next to the date of my comment (8:30 Dec. 27) there's a trash can icon. why? my guess is that it is there to allow the author of a comment to delete the comment but i don't want to test my 'theory' (as Thomas Huxley said "The great tragedy of science - the slaying of a beautiful hypothesis by an ugly fact.") because it mentions the book "Quantum Space" which i think your readers will want to read (at least until/unless you review it negatively).

    btw - i hope you are able to keep in good cheer while dealing with comments on your blog entries. i found that the most difficult part of dealing with the review process involved in publishing a book or article was reading and responding to inane and/or irrelevant reviews and ad hominem criticism. The concept of 'peer review' is a myth since your scientific peers are either your competitors or supporters. the objective evaluation of ideas is a yet another myth perpetrated by the scientific community on the public. it's depressing to think of all of the self-promotional lies our community engages in on a regular basis.

    and finally, this advertisement seems somewhat relevant to the issue of 'theory vs experiment'. (it's not but i've always liked it anyway).

    https://www.youtube.com/watch?v=ijVijP-CDVI

    naive theorist

    ReplyDelete
  68. The Emperor has no super symmetrical clothes.

    ReplyDelete
  69. naive theorist,

    Yes, you can delete your own comments. If you do this, there will remain a note saying "This comment has been deleted by the author." I can then delete it entirely. (Some people use this to correct spelling mistakes since blogger, unfortunately, still doesn't have version-tracking.)

    I am not remotely surprised that particle physicists now try to discredit me, but it only makes their situation worse. Their field has big problems and that's easy to document by the large number of failed predictions - as I lay out in my book (or in the above blogpost). Trying to ignore me will not help them. I'm the messenger, not the message.

    ReplyDelete
  70. It seems to be a self-amplifying process: there is no alternativ to the option to build bigger and bigger colliders. The only "alternative" is to build nothing new - and the money will percolate somewhere.

    My suggestion: take, say, 1% of the sum we would need for a new, bigger collider and make a lotterie in which young, fresh students can win a budget for an research project - but they will get the money not till 10 years later. So they have time enough to think about their projects before start.

    And it has to be fresh students! They should not be affected a lot by orthodox thinking of the established community.

    ReplyDelete
  71. @VYT An electron-positron collider would in some ways be cleaner in that electrons are not composed of internal particles, at least not within the TeV scales we probe. Protons are complicated things with three quarks and a gemish of gluons. The problem with a circular collider for electrons and positrons however is that Brehmsstralung has a power spectrum proportional to m^{-3/2}. As a result an LHC accelerating electrons and positrons would have about 80,000 times the synchrotron radiation (relativistic Brehmsstralung) produced that would be a huge energy loss and could also be a damaging hazard requiring shielding. Electron-positron machines have to be linear, such as SLAC, which means they are given a "one shot" acceleration.

    @ neo. A big upgrade on magnets and RF cavities makes sense for a small multiplier over 13TeV. There are of course a lot of engineering problems, such as with sudden heating of supercooled fluids and material responses to large magnetic fields. This type of upgrade is what happened to LEP.

    @ Axil: It was an interesting result. They quoted a fairly high sigma to the data, though I am not sure how that was derived. I have not heard so far any major trumpets sounding over finding a supersymmetric partner to the tauon.

    Particle physics has really just scratched the surface. We are bumping along at 10 TeV and physics likely has structures up to 10^{16}TeV. The standard model has however given us information on how Ginsburg-Landau phase structure manifested in the Higgs field operates in particle physics at high energy. G-L physics is also prevalent in condensed matter and low temperature physics, and even in phase transitions of coherent laser states of light. So we have some sense of the universality of this type of physics. There may be a hierarchy of Higgs type fields at much higher energy.

    While the LHC has given us no data on supersymmetry I have to stress this does not so much rule out supersymmetry, but rather the supersymmetric extension of the standard model or MSSM. People are still holding out for MSSM, but it is looking very dubious right now. There are a lot of people singing the blues these days as they see their theoretical work potentially founder. There is nothing in supersymmetry that says it must operate at the TeV scale with the standard model. I think this comes down to the prejudice I heard voiced that if you lose your keys in the dark you look at least first under the streetlamp because there is light there. The idea of low mass supersymmetric partners is an assumption built on the bias we have for what can be mustered with particle accelerators. Nature does not care about this ultimately.

    Supersymmetry is a fascinating hypothesis for it connects quantum statistics with spacetime symmetry. A superfield is built around a field, say a boson or fermion, where there are corresponding fermions or bosons with these connected by Grassmann terms. It has an interesting feature very similar to entanglement. Raamsdonk illustrated how spacetime can be thought of as an emergence from entanglement. There is to my mind the possibility that supersymmetry and the quotient space and geometry of entanglement are a manifestation of a general system for nonlocality of quantum states. We expect quantum gravitation to be nonlocal, and it would not surprise me if SUSY and the emergence of spacetime are aspects of a single physics. If so then supersymmetry may be something that holds at much higher energy than the LHC. As I said, we have only scratched the surface.

    ReplyDelete
  72. One thing that has surprised me about the physics community is the ad hominem attacks. The ironic part is that those same ad hominem attackers will at the same time espouse the scientific method. Huh? The community needs some serious training on how to engage in civil and constructive discourse. Furthermore, the community should eject persistent a-holes, like that angry drunken nut from Cern who was shouting nonsense at Bee recently.

    ReplyDelete
  73. @Uncle Al,

    Dear Uncle Al,

    Would you be so kind to include a one sentence high level summary of your point when you make a comment? I would like to understand what you are saying, or even have a vague idea, but usually it is so deep, it is inaccessible to me. Thank you for your consideration.

    ReplyDelete
  74. Sabine wrote: I'm the messenger, not the message.

    I'm learning something just from seeing the responses from physicists who disagree with you. I didn't expect such emotion from high-level physicists.

    In a sense it's a bit humorous, but I feel sad and disappointed. In general scientists are my heroes, but I'm seeing feet of clay. Of course I can't expect scientists to be perfect, but they're supposed to be better than most people at admitting when they might be wrong, when the evidence points that way. They're supposed to be better at guarding against bias caused by self-interest.

    ReplyDelete
  75. I have tons of popular science books from the last 15 years. All books from the lumiaries out there in the field. Lisa Simpson, Brian Yellow et al. They all made great careers on nothingness. Today the books read like bad fairy tales in physics. I want my money back! (^_-)

    ReplyDelete
  76. Sabine

    I can only speak for myself. I tend to see speculative claims as background noise which can and will fade. The results on the Higgs and other precision SM tests at the LHC will, however, stand the test of time. The LHC was not designed as a SUSY discovery machine. We sold it correctly as a machine to elucidate the EW sector and find or falsify the SM Higgs. The funding agencies and politicians bought the idea and we delivered on the promises. Searches were also part of the program but never more than in a subsidiary role. It is unfortunate that a lot of speculation accompanied the LHC. However, a lot of this noise IMO simply filled the vacuum left behind by the delay in the LHC start.

    You worry about promotional videos. However, funding bodies see through hype and look at deliverables. A topical example of this is the ILC. The ILC and its predecessor proposed projects were originally intended to be facilities studying in detail whatever the LHC discovered. As the LHC results confirmed the SM, the ILC was forced to descope from a 500-1000 GeV machine to a 250 GeV Higgs factor (effectively e+e--> ZH) to focus on the real world measurements it would make; the speculative stuff was very much demoted in the light of the LHC results. Now it looks like the ILC won't get funded at all, if the recommendations of the Science Council of Japan (the body advising the Japanese government) are followed.

    Regarding your point about speaking up, its not for me to tell the theorists what they should be thinking about and how they should be working. I know too little to be able to speak with credibility and knowledge. Similarly, its not for the theorists to tell me what to do. They may try and sell the idea of new experiments which interest them but its a general rule in science that proposals with a disproportionate number of theorists on the proposals tend not to be funded.










    I am telling you, your field has to clean up its business. If the head of Fermilab (who, for all I can tell, is an experimentalist) wrongly tells the BBC that we know there must be new science, I expect that particle physicists correct this statement. Yet, no one said a word.

    If the FCC working group at CERN puts out a video that, once again, makes the same false promises, I expect that particle physicists object and ask that the video be taken down. What's the reaction that I actually did get from particle physicists? They think it's okay to lie to the public. (And publicly say so.)

    I cannot recommend to finance people in this field as long as this is going on.

    ReplyDelete
  77. Summerisle,

    Yes, the scientific case for the LHC was the Higgs - as I also explain in my book. But it doesn't matter just who promised what because most people don't distinguish this particle physicist from that particle physicist. They only see that particle physicists "predict" new particles all the time and those never show up.

    The situation for the ILC doesn't look good because the LHC hasn't seen anything besides the Higgs and that fact is in everybody's face now. Same problem for any LHC follow-up, hence the title of this blogpost. What worries me about false advertisement is not so much that people believe it, but that they don't and therefore - correctly - conclude that scientists can't be trusted.

    "Regarding your point about speaking up, its not for me to tell the theorists what they should be thinking about and how they should be working. I know too little to be able to speak with credibility and knowledge."

    That's a lame excuse. You don't need a PhD in physics to see that you've raised a generations of theorists who make career with inventing things that no one sees. If you don't know what I am talking about, read my book. It's in your own interest. Their reputation is your reputation too.

    ReplyDelete

  78. Sabine

    In your opinion its "a lame excuse". A generation of theorists postulated particles which don't exist. In the absence of any experimental clues I really don't know what they should have been doing other. We're pretty much in an unprecedented situation with no unexpected physics for 50 years. Theorists rely on clues from experiment and we haven't been able to give them this. Acknowledging my ignorance is a good reason not to pontificate to others and is far from being a lame excuse. I speak out when I know I have the knowledge and expertise to make a sustained argument.

    You worry about false advertising. As I pointed out to you, and gave the examples of the LHC and ILC, the funding agencies couldn't care less about a promotional video. If the physics case isn't there a collider simply won't be funded (sometimes a good physics case is there like the SSC, ILC 250 GeV and its still not funded). No amount of optimism or hype in a video will change that.

    To get a major new collider approved there are a number of necessary but not sufficient criteria. Some of them are :

    (1) A consensus within the field. At the moment I don't see such a consensus for, eg, the FCC. One may certainly appear but its not there right now. To get a consensus amongst experts, solid quantitative arguments are needed. We don't have a "no lose" theorem for a collider beyond the LHC - this is clear and is no secret. Its also by no means a disaster but it does mean focusing on blue sky exploration, precision measurements and targeted searches. The case must be water-tight concerning what can and can't be done and what can be learned from such a collider.
    (2) Support from other disciplines inside and outside the physics community or, at the very least, an absence of open hostility. Scientists outside the field are very aware of the LHC and that it hasn't seen physics beyond the Standard Model. They are no fools and will not be swayed by a PR campaign when use of the communal funding pot is concerned.
    (3) Government support. Here, a promotional video or two may be of use. However, if a project passes (1) and (2) then the arguments will be sufficiently robust that I doubt you'll have much to complain about with the future videos.












    ReplyDelete
  79. Steven Mason (12:10pm December 29)

    "Of course I can't expect scientists to be perfect, but they're supposed to be better than most people at admitting when they might be wrong, when the evidence points that way. They're supposed to be better at guarding against bias caused by self-interest."

    ROFL.

    naive theorist

    ReplyDelete
  80. @Lawrence Crowell
    From wiki - "LEP collider energy eventually topped at 209 GeV at the end in 2000." I am looking for an answer if this 209 GeV will be enough to generate electron-positron pairs in a collision of two electron beams (instead of positron and electron beams). If that is enough the collision of electron beams will be the control experiment to confirm the SM conclusion about generation of positively and negatively charged particles pairs. So, once again, from SM such pairs should be generated in collision of two electron beams at certain energy, and this should be experimentally confirmed. If no pairs will be detected - SM will have a problem, a big problem. Till this control experiment is done SM is under the question. In any field of science controls are important and for the referee the missing control is the base to reject a publication.

    ReplyDelete
  81. I visited the CERN several times because of personal interest (My professional field is electrodynamics). First time, on the construction site of the LHC I heard dozens of superlatives. They had to dig the biggest hole in loose earth. Every site manager have had at least one heart attack. When they will cooling down the magnets, the street between Lyon and Geneva has to be allocated exclusively for the nitrogen road tankers for weeks and so on.
    So I started to read some books about the physics that warranted such high levels of investment into the infrastructure. And...I was very impressed... until I have read about the grandma of Brian Green and other stories. About string theory I learned nothing essential. The thing I understood was, that only top brilliant scientists are able to catch a little piece of the multiple dimensions behind our reality and that this knowledge justify every investment.
    Some years later, the worst case superlatives on the construction site had not happened. When they measured neutrinos moving faster than the speed of light, my first thought was: check the cables. That was also the moment, I started doubting about the hole project.
    There are many experiments at CERN worth to pursue (For example anti matter investigations). But most of those experiments don't need a super collider.
    And now, after reading Sabines book, I say it again: check the cables (in your theories). Maybe some of the most ingenious inspirations in physics has kinks in the cable.

    ReplyDelete
  82. Summerisle,

    I'm telling you that unless you sort out the business with your theoreticians cooking up nonsense models in the thousands, you'll not see a new collider.

    " In the absence of any experimental clues I really don't know what they should have been doing other."

    If you use a method for theory-development that clearly doesn't work, you should try to correct your mistakes. That hasn't happened and it's still not happening.

    ReplyDelete
  83. Sometimes big new machines get built not because of some physics motivation, but because of inertia. Inertia in thinking, but also inertia, because there is a big lab with a lot of infrastructure (and people) that one wants to preserve. You have an example close by: FAIR in Germany is being built at a price tag of now about 1.5 billion $.

    ReplyDelete
  84. naïve theorist wrote: ROFL.

    I hope it wasn't *that* funny. :-)

    I am genuinely surprised at some of the things I've seen in the blog.

    ReplyDelete
  85. @VYT,

    The Wikipedia site on LEP has the following:

    However, the energy of the bunches is limited due to losses from synchrotron radiation. In linear colliders, particles move in a straight line and therefore do not suffer from synchrotron radiation, but bunches cannot be re-used and it is therefore more challenging to collect large amounts of data.

    Which about says it all. Brehmsstranlung scales as γ^4 with γ = 1/sqrt{1 - (v/c)^2} or γ = E/mc^2, which for LEP was about γ = 50. For the LHC it is γ = 1300, which means from energy an e-p LHC scale machine would produce about a half million times the sychrotron radiation as LEP. OUCH!

    ReplyDelete
  86. Summerisle wrote: If the physics case isn't there a collider simply won't be funded . . . I don't see such a consensus . . . Theorists rely on clues from experiment . . . Acknowledging my ignorance is a good reason not to pontificate to others

    I'm trying to get a fix on your position. Maybe you think a new collider is justified because it might give theorists clues. You're not concerned about marketing hype (or lies) because the physics community can't be fooled. You have faith that the physics community, via a robust consensus process, makes sensible decisions about large, expensive projects like colliders. You also seem unconcerned about the "nonsense" Sabine points to in the physics community.

    Is that a fair assessment?

    ReplyDelete
  87. Thanks for the suggestion above of "Quantum Space: Loop Quantum Gravity and the Search for the Structure of Space, Time, and the Universe" (by Jim Baggott).

    Here is a research report I found interesting:

    "Numerical studies using HPC reveal the existence of an effective space-time description that sheds important light on the way continuum space-time emerges from quantum geometry and potentially links LQG with astronomical observations."
    Glimpses of Space-Time Beyond the Singularities Using Supercomputers
    Parampreet Singh
    [ https://arxiv.org/abs/1809.01747 ]

    I don't know if we need bigger supercolliders. Maybe bigger supercomputers though. :)

    ReplyDelete
  88. Bee,

    any chance you can blog either her or video blog on white holes, LQG white holes, white holes as dark matter as picked up on the press

    for example,

    https://www.newscientist.com/article/mg24032080-100-if-you-think-black-holes-are-strange-white-holes-will-blow-your-mind/

    offering a dark matter explanation via gravity rather than extensions of the standard model. plus white holes in LQG offer answers to the black hole information paradox, and even the big bang - which was a white hole.

    ReplyDelete
  89. Well, Sabine, you "explained in my book that naturalness arguments are not mathematically sound and one shouldn’t have trusted them." I, for one, have never trusted any of them. Rather, I trust important first-principles concepts like Consistency, Causality and One-to-one+ONTO-ness. These important properties of a theory are uniquely achieved ... some are achieved by the No-Boundary WaveFunction, some by (limp) string theory.

    Your essay and book give cause to consider what is generally meant by "naturalness".. but I suspect that its often the simplest mathematical extension. I once had to investigate what is meant by the Cosmological Coincidence, simply to better understand/explain how to modify GR to eliminate it.

    Wayne

    ReplyDelete
  90. Bee,

    Would you be so kind as to illuminate your followers about the prospects within the next decade for high energy particle experimentation with alernate methods, such as lasers? The material I have read is confusing. Some say it will advance rapidly and others say it wil not reach or exceed LHC. What is your view? Would it make sense to hold out for less expensive technology to explore high energy particle physics?

    Thank you,

    Kram

    ReplyDelete
  91. @VYT: The LEP collides electrons and positrons because you can arrange the same magnets to accelerate electrons and positrons in opposite directions around the same circle. To get electrons colliding with electrons, I think you would have to use another separate circular collider.

    ReplyDelete
  92. neo,

    I wrote about white holes here. Tbh, I am presently pretty tired of writing about nonsense. Sorry if that sounds depressing, but that's what it is. It's all nonsense. Maybe I should write about material science for a while.

    ReplyDelete
  93. Kram,

    I am a theoretical physicist. I am not the right person to make predictions about the pace of technological advancement. Let me just say that this is generally a problem with planning missions that take decades to build and complete: By the time you've built the thing, the technology is already outdated. There is nothing you can do about it (unless you want to stop technological progress that is).

    I don't know what you are referring to, but I am guessing it's wakefield acceleration, see here. It looks to me like presently the technology isn't there to build a collider that would exceed LHC energies.

    I doubt it would make it much less expensive though.

    ReplyDelete
  94. Quant,

    Yes, indeed. And if it hadn't been for the theorists with their wild predictions, maybe HEP would just have been carried along by inertia.

    ReplyDelete
  95. @Peter Shor
    From this link to the LEP collider layout - https://www.researchgate.net/figure/The-LEP-collider-layout-and-the-four-LEP-experiments-ALEPH-DELPHI-L3-and-OPAL-at-CERN_fig3_41217049
    I think, there were two separate parallel circles, for electron and positron beams. It should not be a problem to inject electrons instead of positron to get electron beams in two circles in opposite direction to collide.

    ReplyDelete
  96. @ neo: I saw this article last week on my i-phone when I logged into Google. The Google algorithm must know I tend to read this sort of thing and it gives me a list of these sorts of articles. I read it and thought it was in a way silly.

    Do white holes exists? Well sort of maybe, but not in the way we might naively think. I use Bee's image on the blog page

    http://backreaction.blogspot.com/2009/11/causal-diagram-of-black-hole.html

    because we can't post images on the blog commentaries. Take a look at the left or right squares labeled A and B. The black hole horizon at r = 2m in both of these is split into a past and a future horizon. The flow of matter is from the bottom to the top and null rays or light follow the diagonal lines. So we can think of this as saying there are white holes with the past horizon with matter and radiation flowing out and a black hole with the future horizon with matter and radiation entering it. The Eddington-Finkelstein diagram is interesting, where the white hole is found by turning it upside down. So does this mean we have these fountains of matter and radiation in the universe? Astronomers have looked for them and so far nothing. A solar mass object that erupts all of its mass into radiation, at least in one burp, would be a stupendous event.

    However, Hawking radiation does give us some idea that maybe the emission of radiation by a black hole is some tiny quantum fluctuation of the black hole into a white hole. A quantum black hole that is emitting a lot of Hawking radiation would appear FAPP as in part a white hole. So what is this diagram telling us? It is really just a mathematical device and it may carry with it signatures of the quantum nature of black holes and spacetime. It is a conformal map of an exact solution of the “eternal” black hole, which can't be entirely physical as black holes are made up of matter that implodes inward. So conformal symmetry is likely broken. However, this does give us some reason to ponder deeply on the quantum nature of spacetime. Ideally the two regions A and B are entangled, the two black holes observed in these regions are entangled and the two triangles for the black hole and white hole may also represent two states or sets of states. I don't think astrophysical black holes are really entangled this way in a pure state, nor do I think there exists some corresponding white hole. However, this may still be indicating something about the relationship between spacetime and quantum mechanics.

    I think all too often physicists look at these mathematical devices, whether string theory or LQG (which is just a system of constraints), or other things and jump immediately to some grand conclusion on the nature of the world. This gets picked up by Scientific American and New Scientists in some article on a great new revolution in physics. This then leads to confusion and disappointment in a few years when that fades and there is the next new thing.

    ReplyDelete
  97. Any theory that fundamentally excludes the visible universe is fundamentally wrong.
    … BARYOGENESIS
    There remains a microwave rotational spectrometer, some crafted organic molecule test masses, and funding commensurate with CERN's daily coffee and doughnut budget. Do it.

    ReplyDelete
  98. @Lawrence Crowell
    "Ideally the two regions A and B are entangled, the two black holes observed in these regions are entangled and the two triangles for the black hole and white hole may also represent two states or sets of states."

    If I understand it correctly, Gerard 't Hoofd thinks that the white hole solution is just the anti-podal opposite part of the same black hole. Matter "falls through" a black hole and re-emerges at the ant-polar opposite end as if the black hole does not exists. Except that the exit happens "delayed", when the black hole evaporates.

    http://www.staff.science.uu.nl/~hooft101/lectures/GtHBlackHole_2018.pdf

    ReplyDelete
  99. @bee @lawrence

    the difference between your post and the current articles is 2 years, as your post was written in 2016.

    in 2018 LQG researchers show how under LQG black to white hole transitions happen.
    A. Ashtekar, J. Olmedo, and P. Singh, “Quantum transfiguration of Kruskal black holes,” Phys. Rev. Lett. 121, 241301 (2018); “Quantum extension of the Kruskal spacetime,” Phys. Rev. D 98, 126003 (2018).

    Rovelli in the article also states that many quantum mechanical microscopic black holes at the big bang could have transformed to white holes of planck mass (1 microgram) and be the dark matter

    ReplyDelete
  100. @ neo, 't Hooft is a smart guy, and I read his paper a couple of years ago. His theory as I remember appears to be almost a double cover of what we usually think. It has been a while since I looked at this though, but it may be pointing to something.

    The biggest thing that strikes me in pondering the similarity between the black/white hole is how similar it is to quantum mechanics. I have thought for some time that general relativity and quantum mechanics are either identical or dual aspects of something more general.

    ReplyDelete
  101. @Lawrence Crowell said “So does this mean we have these fountains of matter and radiation in the universe? Astronomers have looked for them and so far nothing.”

    The only thing I see that looks like a fountain is black hole jets. There is a recent relativistic jets survey on arxiv (1812.06025v1) that says accretion disks at the equator somehow make it to the poles and are jetted. To quote:

    “AGN jets are formed when the black hole spins and the accretion disk is strongly magnetized, perhaps on account of gas accreting at high latitude beyond the black hole sphere of influence.”

    Yeah, that sounds sketchy.

    So, I get the no hair theorem, but is science really certain that the jets are NOT coming from inside the SMBH?

    ReplyDelete
  102. I worked in software development, for 30 years. It was well known that really big projects can end up chasing a receding market window, or bog down in complexity, and ultimately be cancelled with much money lost. In fact, industry pundits published interesting statistics on the relationship between the 'size' of a project (in terms of number of developers, lines of code, budget, and other dubious metrics) and the likelihood of completion. And those curves were brutal - for very large projects, the chance of success becomes small.

    Might the LHC break the business model of particle physics? I saw firsthand how the failure of a huge software project - after years of effort and many millions of dollars spent - can so traumatize a big company that it basically resolves never to launch another. And the careers of those who approved and managed the failing project were dead-ended.

    I have to wonder if a similar relationship exists between the 'size' of a physics experiment and the likelihood that it ever delivers results that justify the costs.

    ReplyDelete
  103. neo,

    Yes, I've seen the paper. It's LQC, not LQG (see the paragraph on "limitations"). Look, let me put it this way: I think it's not interesting enough to spend my time on.

    ReplyDelete
  104. @ neo: This paper by Ashtekar, Olemdo and Singh (AOS) sent to PRL is short and more to the point:

    https://arxiv.org/abs/1806.00648

    than their other sent to PRD. This does have some elements of what I mention with the black/white hole containing quantum-like structure. This paper even enters into discussions on white holes. AOS though lean heavily on the LQG construction. I think honestly this points to something different entirely.

    Maybe I am wrong, but I think this is one point to Bee's objection to current theoretical physics. We tend to develop mathematical techniques, or sometimes I think of as gadgets, and in doing so think we have a new paradigm --- to use a worn out term from Kuhn's worn out thesis. However, if you think about it the development of special relativity did not really involve any deep mathematics, but rather a new physical idea. The same holds for the early development of quantum mechanics. In a sense to rethink the very thinks, thoughts and thunks of physics really requires being almost a sort of philosopher willing to ponder things. In my case this is often on walks or on horseback in the woods with my dogs following, outside the hustle of academic physics departments.

    @ Kram Sirrom: The business of hydrodynamics in curve space and spacetime was pioneered by Taub. Hydrodynamics is based around the Navier-Stokes equation, which is nonlinear and so far its solution set for ordinary fluids in flat space is not completely known. Now throw electromagnetism into the mix to get magnetohydrodynamics of plasmas and you are really fighting a dragon. Or one can look at fluid motion in curved spacetime and you have a third dragon. Now put them all together and you are fighting a three headed dragon or Cerberus. With computers and grid adaptive algorithms though these problems are starting to be at least somewhat tamed. There have been lately recent announcements that the dynamics of plasmas around supermassive black holes at galaxy centers, such as the more modest one in the Milky Way center Sagittarius A, instead of accretion disks have torus or tubular flows of hot gas around them. There is a big effort with radio astronomy to image the black emptiness of the BH horizon.

    Many years ago on my 286 machine I wrote a FORTRAN code that looked at the motion of a charged particle around a massive body (planet or star) with a magnetic field. The motion of this particle for a charge to mass ratio large enough but not too large was along spirals situated over the poles of the main body. This turned out to be some physics behind jets, where if charged particles are given enough of an energy kick they can exit along the poles. Also atoms in extreme magnetic fields have electron orbitals that enter into tight regions so the atom becomes a bit like a string.

    I would employ the method of Sherlock Holmes, which is you investigate the most probable causal process for solving a problem. If that is eliminated then you go to the next most probable causal explanation. This is kept up until you find the explanation that works. With jets from quasars and active galactic nuclei (AGN) I think the more ordinary hydrodynamic explanation is likely to win the day. The idea that black holes in AGNs are flipped into white holes is far more extraordinary.

    ReplyDelete
  105. Jim_h wrote:

    "I have to wonder if a similar relationship exists between the 'size' of a physics experiment and the likelihood that it ever delivers results that justify the costs. "

    Having worked in software for most of my life after my PhD in Chemistry, I think Jim's comment is spot on! Not only do I think such projects run away with themselves on cost, but without a sequence of intermediate deliverables, I wouldn't trust the end result even if it appeared! Thus I don't trust that the Higgs boson, or any of the other particles detected in a similar way (i.e. purely by analysing the decay products), were real discoveries at all!

    Science has, after all, thrived on reproducibility. Nobody is going to check the Higgs can really be generated at a mass of approximately 125 GeV

    Think of the 64-bit Windows 10 operating system. That exists now because a far simpler operating system was created (early DOS version), and progressively modified to incorporate windowing in a very kludgy way in Windows 3, then a less kludgy Windows XP, then came a change in instruction set to use 32-bit protected mode, then, after a number of more steps cam the transition to a 64-bit instruction set, and a few more steps gave us 64-bit Windows 10 (I have missed out many, many steps). Each step was a saleable product that got debugged in actual use by people all over the world. Any attempt to create an operating system like Windows 10 in one go, would almost certainly fail.

    ReplyDelete
  106. "in 2018 LQG researchers show how under LQG black to white hole transitions happen."
    There then must be 10+ solar masses' energy-equivalent flashes whose spectra include no atomic transition lines and no time evolution. There are none. If the flash is not spatially isotropic, thrust.

    Not nearly everything in a universe degenerates into black holes - including black hole binding energy and fields in general. The cycle quickly damps. The model is inherently defective, and vigorously excludes baryogenesis.

    ReplyDelete
  107. Maybe it is obvious remark, but I have found it funny when it comes in my mind: we don't need larger collider, we desperate need smarter physics :)

    ReplyDelete
  108. @Bee,
    ok.

    there are recent papers on both f(R) gravity and conformal gravity reproducing MOND phenomenology. does this interest you?

    I hope for 2019 you create youtube videos on LQG, LQC, f(R) gravity and conformal gravity, asymptotic safe gravity etc. like you did for strings and dark matter.

    re: "How the LHC may spell the end of particle physics"

    the news reports I've seen suggest that CERN future circular collider or China will build a 100TEV 100km scale collider, by 2050. granted a lot of things can happen by 2050, as the US Superconducting Super Collider was cancelled. right now though plans are in the works for a new post-LHC pp collider.

    btw if a 100km tunnel either CERN or China plans with 16 tesla magnets gives you 100TEV collision energies, does it follow a 200km tunnel with same 16 tesla magents give 200 TEV collsion? if so why not just dig 200km tunnel.

    ReplyDelete
  109. The relative lack of interest among both experimentalists and theorists in advanced accelerator technology research (passing mentions by Lawrence and Bee, no mention by experimentalists Don and Summerwise) is telling. The late Burton Richter believed that working on wakefield and other advanced concepts was the sensible way to proceed on the energy frontier, and to anyone thinking about R&D economics the idea of proceeding with a decabillion-plus scaled-up conventional accelerator before spending even a billion dollars on advanced concepts is foolhardy. Yes, the new technology is risky and uncertain and pursuing it first would delay breaking ground on any conventional mammoths. A whole community of conventional device builders and operators would have limited employment during this research, and if it were successful much of their expertise would become obsolete. Neither is a good reason for putting 95+% of resources into work on scaled-up conventional technology.

    ReplyDelete
  110. Richter from 2014, well worth reading on the entire situation:
    https://arxiv.org/pdf/1409.1196.pdf

    ReplyDelete
  111. @ David Bailey

    Two comments about this:

    1. One of the ways that the LHC was calibrated was by essentially reproducing many previous major particle physics discoveries: this is a pretty good 'intermediate deliverable.' See, for example, https://cornellmath.wordpress.com/2009/12/01/rediscovering-the-standard-model/ or https://atlas.cern/updates/atlas-blog/art-rediscovery

    2. You are right, in general, that reproducibility is a concern when trying to make discoveries using instruments with unique capabilities. However, independent evidence for the Higgs consistent with the LHC results was also seen at the Tevatron, albeit not at the statistical significance required to claim an independent discovery: https://www.nature.com/news/boost-for-higgs-from-tevatron-data-1.10167 . Furthermore, while the two detectors at the LHC, CMS and ATLAS, share a beamline, they also differ in many ways and are operated and analyzed by separate teams, providing a degree of independent confirmation within the LHC.

    In general it is wise to ask questions about whether we can really trust these results, but naive to imagine that the people dedicating their lives to this haven't asked similar questions and at least come up with some attempts to deal with them.

    (Note that I am not a high-energy physicist, and the above views can be taken as those of a layman)

    ReplyDelete
  112. So what's wrong with the standard model? Maybe there's some deeper connections
    orders of magnitude away, but you just have to wait for the data. Outside the
    SM you've got netrino mass, dark matter and dark energy (lamda?) Those could all
    be some weird gravity effect... like if the graviton has some really small non-zero
    mass. (I'm a dilettante, but got Fenyman's gravity book for xmas.. bogged down/ lost
    in chapter 3, 4.)

    George H.

    ReplyDelete
  113. I am looking for an answer if this 209 GeV will be enough to generate electron-positron pairs in a collision of two electron beams (instead of positron and electron beams). If that is enough the collision of electron beams will be the control experiment to confirm the SM conclusion about generation of positively and negatively charged particles pairs. So, once again, from SM such pairs should be generated in collision of two electron beams at certain energy, and this should be experimentally confirmed. If no pairs will be detected - SM will have a problem, a big problem. Till this control experiment is done SM is under the question. In any field of science controls are important and for the referee the missing control is the base to reject a publication.

    Yes, controls are important. However, if electron-electron collisions did not result in pair production at high-enough energies, there would be a lot more in trouble than just the SM. Of all the things one could do, considering that funds are limited, building a high-energy collider to see pair production from electron collisions has to be pretty low down on the list.

    No, the SM is not under question or in trouble because this experiment hasn't yet been performed.

    The standard model also says that the cross section for various interactions doesn't depend on the day of the week, or how many people are in London at the moment, or whatever. I believe them, despite the lack of control experiments.

    ReplyDelete
  114. PhDStudent said:

    "(Note that I am not a high-energy physicist, and the above views can be taken as those of a layman)"

    Thanks for your comment, which certainly gives some reassurance that the LHC is reporting something real.

    ReplyDelete
  115. You seem to be up on Black hole stuff. Can you explain this point that is confusing me?

    Hawking radiation produces a pair of particles created just outside the event horizon of a black hole. It is possible that the positive member of the pair (say, an electron) may escape - observed as thermal radiation emitting from the black hole - while the negative particle (say, a positron, with its negative energy and negative mass) may fall back into the black hole, and in this way the black hole would gradually lose mass. But a positron has positive energy(512KeV). Can you explain how a particle with negative mass converts itself into a particle that has positive energy? Does it involve the flow of time? Have you already covered this in your blog?

    ReplyDelete
  116. @ Axil: Remember that this picture of an electron pair is a bit of a heuristic. However, we can address this matter from the perspective of quantum field theory and Dirac's old idea of the “Dirac sea.” To start Dirac proposed that the anti-electron sat in the bottom half of the momentum-energy light cone. The Lorentzian light cone with time arrow inside the cone and the basis vectors for spatial directions outside has its Fourier transformed version with energy direction inside the cones and momentum basis vectors out. The negative energy bottom half cone is completely filled, and if it is not then some positive energy will fall in and fill it emitting a photon. The positive top part of the cone has positive mass-energy electron states. The two halves of the cone have a gap of mc^2 for the mass of the electron. So to generate an anti-electron one must impart 2mc^2 or more energy into a state in the negative half of the cone to jump it into the positive energy cone across the mass gaps. This means the physical anti-electron has a positive mass. Along with this positron comes an electron as well. By pulling a positive charged negative energy state in the bottom light cone you have left a “hole” and this has a negative charge in this “sea.” That is canceled by generating a negative charged electron with positive mass and the hole is filled..

    We now can use this to think of more modern quantum field theory. The electron and positron were pulled out of the vacuum by imparting sufficient energy in some interaction, such as a gamma ray scattering off a nucleus. These loops or virtual particles in the vacuum can be removed by a process called normal ordering. We expect these processes to have no contribution unless they are coupled to real particles. In that case these things are summed over for a range of energy or momentum and as such are considered “off shell.” So we have no particular trouble thinking of the particle that enters the black hole as negative and the real particle makes it out of asymptotic infinity. However, this picture becomes a bit problematic when we consider gravity. The vacuum may be removed for ordinary quantum fields, but with gravity it will still have some gravitational interaction.

    Now consider the Penrose diagram for the Schwarzshild metric. The horizons for the X in the middle and we then think of there being a quantum loop, such as for the e-p pair at the center. Gravitation determines the path of a particle classically as following geodesic flows. However, if we think about the path integral of quantum mechanics only one path for a particle is on the geodesic; the other paths are really nongeodesic. The same holds for this loop. I wrote on Stack Exchange on how the Kerr-Newman black hole for a stationary accelerated frame near the black hole the local spacetime region in that frame is AdS_2×S^2, where AdS_2 is an anti-de Sitter spacetime in two dimensions.

    https://physics.stackexchange.com/questions/262735/ads-black-holes/262744#262744

    The AdS spacetime has negative curvature or a negative vacuum energy. That observer will only be able to detect part of that loop, as this observer never crosses the horizon. This deformation of the vacuum into a locally negative value is one way within spacetime that we can think of the black hole as absorbing negative mass-energy. That loop is also on a non-geodesic path and the result is that negative mass-energy crosses the horizon. Then true to the old idea of Dirac positive energy particles emerge from the black hole.

    ReplyDelete
  117. Hi Sabine,

    I think you are absolutely correct in your judgement that policy makers will judge particle physicists as dishonest and that in any case, no utility either ongoing with the LHC or more powerful colliders is to be expected. However, I feel that physicists are getting off the hook far too easily. The reality is that not only should new colliders not be funded, but that the existing LHC should be defunded and the Government policy should be angled towards abandonment and having physicists reapplied to more fruitful fields. I really believe the LHC will benefit society as a museum piece rather than a continuing self fulfilling fantasy that they could actually discover anything of use. The application of (supposedly) intelligent physicists onto the development aspect of research and development will benefit the world far more than they imagine foundational discoveries could do.


    Curiosity about anomalies, and doubts about the mainstream scientific views are the seeds of new foundational discoveries- not stepwise increases in power of experiments and complexity of explanations. A defunding of current high energy particle physics experiments will stir the pot enough to encourage a move towards research that will be more fruitful. I am hoping that Governments pull back the purse strings rather than defer to supposedly stronger intellect.

    That is not to mean that physicists will lose their jobs - The money will be available from the Governments concerned. Just reapplied away from even ongoing mega-projects.


    Regards

    Marco

    ReplyDelete
  118. @Plasmoid Anomaly,

    How right you are!

    If you were to imagine a worldline where there is a tremendous unifying 150 year breakthrough in the next month, what then?

    Let’s further imagine that the new theory is simple, elegant, natural, beautiful, symmetrical, and requires no math beyond Maxwell’s equations.

    Such a turn of events - I would be very interested in Bee’s analysis post facto.

    How would you organize the people and process to reswizzle physics and cosmology in light of such a 150 year breakthrough?



    ReplyDelete
  119. How can physicists get out of the crisis of understanding in the basis of fundamental science, overcome the “troubles with physics” (Lee Smolin) and the “loss of certainty” (Morris Kline)?
    Theoretical physicists should always remember the John Archibald Wheeler covenant:"Philosophy is too important to be left to the philosophers."
    The crisis in the philosophical basis of knowledge is an ontology crisis. We need new super "crazy" ontological ideas. But unfortunately, ontology is not interesting to physicists.
    "An educated people without a metaphysics is like a richly decorated temple without a holy of holies." (G.Hegel)

    ReplyDelete
  120. As someone who has had nearly six decades of exposure to the UFO literature, I do have a glimmer of hope that there is new physics associated with gravity, or inertia, well below the Planck scale. An extraordinary report, such as the April 22nd, 1966 Beverly, Massachusetts incident, is a case in point. Of course that word "extraordinary" reminds us of Carl Sagan's dictum: "extraordinary claims require extraordinary proof". However, within a subject flooded with images of sun dogs and spiraling rocket exhausts in the thin upper atmosphere, this one stands out. It involved multiple witnesses; two Beverly police officers called to the scene, three women, including the mother of the 11 year old girl, who was the initial eyewitness. It was claimed that three, silent, "automobile-sized", "plate shaped" objects circled, at low elevation, over Beverly High school and its adjacent grounds. One of the objects reportedly came with 20 of an eyewitness, directly overhead.

    Granted, it's a leap of faith, but if we take this account along with other reports exhibiting similar characteristics, at face value, it's tempting to propose that our global society is bearing witness to products of a foreign, non-terrestrial technology (or technologies) auguring new physics that transcend our contemporary scientific knowledge. And, as we do not see galactic size particle accelerators, it may not be necessary to probe all the way to the Planck energy scale before new physics is uncovered.

    ReplyDelete
  121. Schroeder wrote: it's tempting to propose that our global society is bearing witness to products of a foreign, non-terrestrial technology

    Besides a proposal to bear witness to aliens, if and when the opportunity arises, what else do you propose?

    I'm not trying to be argumentative, but if I see an alien, I don't need anyone proposing that I should bear witness to an alien. When someone sees something extraordinary, it tends to grab their attention.

    Let's hope that the aliens aren't Kanamits. :-)

    ReplyDelete
  122. @Steven Mason: Well, I learned a new word that I never heard before "Kanamit" - presumably stimulated by the 1950's sci-fi classic "The Day the Earth Stood Still", where the 9 foot tall robotic alien "Gort" emerges from a landed craft in Washington, DC. I should have used the word "conclude" instead of "propose" to make the sentence logically coherent.

    By the way in case anyone is interested here is a good synopsis of the incident I referred to, assuming it's OK to post this link: https://ufologie.patrickgross.org/htm/beverly66.htm

    ReplyDelete
  123. @Don Lincoln: "Personally, I was hoping for something to explain the generation problem...perhaps compositeness or some other equally disreputable idea that theorists didn't like." It's interesting that you were thinking along the lines of compositeness with respect to the three generation puzzle. Back a quarter century ago a seemingly simple and elegant idea occurred to me to account for why there are three particle families, and compositeness was an integral part of it. Getting quite excited I sent off copies of a paper describing the idea to various magazines and to at least one professional journal. Not too unexpectedly no one would consider publishing it. In the paper I basically glossed over the consequences that compositeness would have brought into the model, which made the whole thing unworkable.

    Just recently I tried to rework the idea without compositeness, and early on it looked quite promising. For example it gave an explanation for why certain non-standard interactions, necessitated by the model, could occur without being noticed in particle tracks. And, earlier, I had already recognized that the model had a built-in explanation for the LNSD and MiniBooNe anomalies. I was giddy with excitement, at the pinnacle of a roller coaster ride, oblivious to the law that whatever goes up must come down again. Then, a careful accounting of weak-isospin in several non-standard interactions showed that they didn't add up properly from input to output. Tried every trick I could think of, but nothing worked. So, alas, a ride that climbed to heights of elation plummeted to a low of dejection, without a stock-market-style plunge-protection team to cushion the fall.

    But on the plus side it was a learning curve. I plan to tinker with the idea some more, to see if there's any work-around the problem, before giving up entirely on it.

    ReplyDelete
  124. @sabine

    "Therefore, it may very well be that the LHC will remain the largest particle collider in human history."

    Wrong prediction, I'm afraid!

    The largest collider will "soon" (read, one decade or so) be built by the chinese, CEPC.
    100 km tunnel lenght.
    In the meantime, in spite of your call to defund research in this field, the international collaborations behind the HL-LHC and FCC-hh programs will have found out how to build 14m-long, 16 T superconducting magnets (or even 20 T), and at that point building SppS, the chinese version of the FCC-hh, will be a matter of another decade or so... the "exorbitant" (as per your mantra) costs of SppS will be a minor fraction of 1% of the GDP of China in 2030-2040.

    All this, of course, provided that there is no chinese version of sabine who writes a book and stops progress in its tracks... :-)

    Cheers.

    ReplyDelete
  125. @Lawrence Crowell

    Lawrence Crowell said...
    @VYT,

    "The Wikipedia site on LEP has the following:

    However, the energy of the bunches is limited due to losses from synchrotron radiation. In linear colliders, particles move in a straight line and therefore do not suffer from synchrotron radiation, but bunches cannot be re-used and it is therefore more challenging to collect large amounts of data.

    Which about says it all. Brehmsstranlung scales as γ^4 with γ = 1/sqrt{1 - (v/c)^2} or γ = E/mc^2, which for LEP was about γ = 50. For the LHC it is γ = 1300, which means from energy an e-p LHC scale machine would produce about a half million times the sychrotron radiation as LEP. OUCH!"

    ???????
    This is completely wrong!
    Bremshstrahlung has nothing to do with SR, other than astrophysicists call it "magnetic bremsstrahlung", but the real bremsstrahlung in accelerators is something completely different, it is inelastic scattering of the particle beam (protons or electrons or something else) against the protons of the residual gas inside the vacuum chamber.

    To make the math straight, the relativistic factor gamma for LEP was MUCH HIGHER than that of LHC, it would be higher even of that of FCC at 50 TeV.
    Gamma for LEP at 104 GeV (highest energy reached in phase 2) was ~ 203500... LHC's at 7 TeV is "only" 7460.... that's because the mass of the emitting particle, the electrons/positrons for LEP is almost 2000 times smaller than that of LHC, the protons.

    Synchrotron radiation total power generated in LHC at 7 TeV and 528 mA current is only 3.5 kW, while LEP-2 at 104 GeV and 4 mA was 13.37 MW.

    Cheers.

    ReplyDelete
  126. @Ueli

    "When they will cooling down the magnets, the street between Lyon and Geneva has to be allocated exclusively for the nitrogen road tankers for weeks and so on. "

    ???
    How many ridiculous, irrealistic things can be written on this blog, devoid of any factual truth?
    What the hell are you talking about... the coold down from room temperature of the "cold mass" of the ~1200 superconducting magnets needs of the order of 10-20 truckloads of liquid nitrogen per day... for many days, of course... but far from the "exclusive allocation" of the street between Lyon and Geneva... that's total nonsense to say the least.

    ReplyDelete
  127. David Schroeder wrote on 12 January 2019

    "...Then, a careful accounting of weak-isospin in several non-standard interactions showed that they didn't add up properly from input to output. Tried every trick I could think of, but nothing worked..."

    I would keep trying as weak isospin eigenvalues do not 'appear' to be conserved in interactions. Take a LH electron emitting a photon and then continuing as a RH electron. Input has weak isospin of -0.5 while output has zero (= 0 + 0) weak isospin. IMO there may possibly be something missing or just covert in the Standard Model version of the input, eg something inputted from the vacuum. Eg weak isospin of 0.5 from a higgs field. (I am merely an amateur so check on this.) But either you have to make w.i. balance in all interactions, or else you don't.

    Austin J. Fearnley

    ReplyDelete
  128. Austin, Thank you informing me about this and the encouragement. I greatly appreciate that. Amazingly, last night, I was on page 113 of the Third Edition of "Introduction to High Energy Physics", by Donald H. Perkins, and right at the bottom of the page, the very interaction that caught my attention back in 1994 - a neutral lambda decaying to a proton and negative pion - and was completely surprised to see that both isospin and weak-isospin weren't conserved in this weak force mediated interaction. Somehow I had gotten it in my head that, at least, weak-isospin must always be conserved.

    I'm also an amateur, (retired engineering technician in oceanography), but had been interested in the field since early teens, inspired by Scientific American articles on the subject. The electroweak synthesis of the 70's, and popular books on it in the 80's rekindled my interest in the field that had languished for a decade or so. I was one of those 'crackpots' that disturbed the peace of some prominent physicists by mailing off theory-ideas between 1991 to 1996. The first of these was completely wrong, but was amended by a far better idea a few years later in 1994.

    So, yes, I'm going to continue working on this idea, which I can't explain here because of (very sensible) blog rules. Am just getting over a 5 day case of the flu. Add to that a shocking car repair bill and breaking a tooth last night, I'm in something of a funk, not able to concentrate too well. But as soon as I get the tooth fixed, and get over the shock of that repair bill, I plan to plunge back into this idea with renewed vigor. Thank you, again, for the encouragement and information.

    ReplyDelete
  129. It's a bit more complicated, but well known; there is even a Wikipedia page on it!

    ReplyDelete
  130. Glad to be of help/encouragement, David. This site could get one down as it seems like one laugh short of an execution warrant :)

    In terms of the blog theme, IMO as an amateur, weak isospin still has a lot in it to be discovered or at least is connected with a lot to be discovered in particles/fields and their interactions, within the Standard Model. Neutrinos, higgs (maybe plural), maybe spontaneous emissions, and fermion mass acquisition. I suspect these and probably more are in need of theoretical answers first. Not sure if the LHC has enough energy to dot the Is and cross the Ts of the Standard Model. I suspect not. OTOH without the theory, proof may already be hiding in plain sight of the LHC.

    ReplyDelete
  131. Austin, I'm in a much better frame of mind today, after a harrowing, 140 mile, round-trip, ride to the dentist with spongy brakes that went almost to the floor, yesterday morning. Got the brake fixed yesterday afternoon (weren't bled properly). Interesting that you brought up multiple Higg's - "Higg's (maybe plural)", as there has been speculation about possible heavier cousins of the W and Z bosons, as well. The model I'm working on requires at least one additional Z-boson to work (and possibly extra W-bosons, as well). Since no additional Z-bosons (or W-bosons, for that matter) have been found, that would seem to negate the model. But in the framework of the model certain standard particle decays would involve more than one stage during the decay sequence. The lifespan of the intermediate stage would be so brief, that it wouldn't be detected, or extremely difficult to detect. It is here that evidence for an additional Z-boson would be hiding.

    Of course the whole concept is entirely speculative, and it might end up being just another amateur-level flash-in-the-pan. But with a more relaxed state of mind after some recent unpleasant experiences, I'm going to do the best I can to put the finishing touches on this model.

    @Phillip Helbig, thanks, I actually printed Wikipedia article out, and keep it with a sheaf of other papers for handy reference.

    ReplyDelete
  132. Austin, I wrote a short post earlier, responding to your latest post in which I broached, in a rather oblique fashion, some details of my personal model-theory. This might be against blog rules even in that format. In any case I kind of mangled what I had intended to say, along with implying that I was close to completing the model, which isn't at all the case. There are just a lot of things to take into account, including some of the excellent ideas you brought up in your 5:30 AM, January 24, 2019 post.

    In that earlier post I meant to say that particular particle decays, in the model, would require two stages to complete (instead of one), with an in-between particle state extremely short-lived, and thus possibly escaping detection. But this scenario could run up against probability problems, even though there are only two weak vertices per interaction in the corresponding Feynman diagrams. There's just a ton of things to consider, and to be honest I don't have as good a grasp of the Electroweak theory as I should to be trying to devise a modification to a theory that is phenomenally well vetted. Coming up with a coherent model-idea has lots of potential pitfalls that someone with my knowledge base can easily overlook. So I need to be extremely careful in putting it all together, before I attempt to publish it.

    ReplyDelete
  133. Please, can someone explain the requirement that it is mass^2 rather than mass that appears in the Higgs term of the Lagrangian? What is it about pure scalar particle that causes this to be true? It seems non-physical. Energy units are [kg-m^2/s^2]. Throwing an extra [kg] into the Higgs term makes the rest of that term ugly, very unnatural. So far I've no success in trying to track this down. Just how technical can it be?

    ReplyDelete
    Replies
    1. The Lagrangian has mass dimension 4, the Higgs-field has 1, meaning you need an m^2 in front of the \phi^2 to get the dimension right. You know the dimension of the Higgs-field from the kinetic term.

      Delete
  134. lol. pretty technical. thank you. Succinct. Helpful. Good search terms. Hope to be able to connect this to my present quandry re what makes pure scalar different. Wavefunction is comprised of scalar, vector, axial vector,... Why quadratic divergence for scalar and not the higher dimensional components? Why no quadratic divergence for the pseudoscalar?

    ReplyDelete

COMMENTS ON THIS BLOG ARE PERMANENTLY CLOSED. You can join the discussion on Patreon.

Note: Only a member of this blog may post a comment.